QUESTIONS PRACTICE MBE — A.M. EXAM

Directions: Each of the questions or incomplete statements below is followed by four suggested answers or comple- tions. You are to choose the best of the stated alternatives. Answer all questions according to the generally accepted view, except where otherwise noted.

For the purposes of this test, you are to assume that Articles 1 and 2 of the Uniform Commercial Code have been adopted. You are also to assume relevant application of Article 9 of the UCC concerning fixtures. The Federal Rules of Evidence are deemed to control. The terms “Constitution,” “constitutional,” and “unconstitutional” refer to the federal Constitution unless indicated to the contrary. You are to assume that there is no applicable statute unless otherwise specified; however, survival actions and claims for wrongful death should be assumed to be available where applicable. You should assume that joint and several liability, with pure

comparative negligence, is the relevant rule unless other- QUESTIONS wise indicated. PRACTICE MBE — A.M. — MBE PRACTICE QUESTIONS PRACTICE MBE — A.M. EXAM

Question 1 (B) Only by an offeree’s making the arrest and assisting The owner of a three-acre tract of land with a small resi- in the successful conviction of an arsonist within the scope of the offer. dence rented it to a tenant at a monthly rental of $200. After (C) By an offeree’s supplying information leading to arrest the tenant had been in possession of the tract for several and conviction of an arsonist within the scope of the years, the tenant and the owner orally agreed that the ten- offer. ant would purchase the tract from the owner for the sum of (D) By an offeree’s communication of assent through the $24,000, payable at the rate of $200 a month for ten years same medium (television) used by the city in making and also would pay the real estate taxes and the expenses its offer. of insuring and maintaining the tract. The owner agreed Question 3 to give the tenant a deed to the tract after fi ve years had passed and $12,000 had been paid on account and to A cigarette maker created and published a magazine adver- accept from the tenant a note secured by a mortgage for tisement that featured a model dressed as a race-car driver the balance. The tenant continued in possession of the tract standing in front of a distinctive race car. In fact, the car and performed his obligations as orally agreed. The ten- looked almost exactly like the very unusually marked one ant, without consulting the owner, made improvements for driven by a famous and popular driver. The driver in the ad which he paid $1,000. When the tenant had paid $12,000, was not identifi ed, and his face was not shown in the adver- he tendered a proper note and mortgage to the owner and tisement. The cigarette maker published the advertisement demanded delivery of the deed as agreed. The owner did without obtaining the famous driver’s permission. - not deny the oral agreement but told the tenant that she car driver sued the cigarette maker for economic loss only, had changed her mind, and she refused to complete the based on common-law misappropriation of the right of pub- transaction. The tenant then brought an action for specifi c licity. The cigarette maker moved to dismiss the complaint. performance. The owner pleaded the Statute of Frauds as Will the cigarette maker’s motion to dismiss the com- her defense. If the owner wins, it will be because plaint be granted? (A) nothing the tenant could have done would have over- (A) No, because there are sufficient indicia of the driver’s come the original absence of a written agreement identity to support a verdict of liability. (B) the actions and payments of the tenant are as consis- (B) Yes, because the driver is a public fi gure. tent with his being a tenant as with an oral contract (C) Yes, because there was no mention of the driver’s (C) the tenant did not secure the owner’s approval for the name in the ad. improvements that he made (D) Yes, because the driver did not claim any emotional or (D) the owner has not received any unconscionable bene- dignitary loss. fi t, and, therefore, the tenant is not entitled to equitable Question 4 relief A man was the illegitimate, unacknowledged child of his Question 2 father, who died intestate, leaving neither spouse nor any During 2010 a series of arsons occurred in a city. In early children other than the man. The state’s law of intestate suc- 2011 the city council adopted this resolution: cession provides that an unacknowledged illegitimate child The city will pay $10,000 for the arrest and conviction of may not inherit his father’s property. The spouse, all other anyone guilty of any of the 2010 arsons committed here. blood relations, and the state are preferred as heirs over the The foregoing was telecast by the city’s sole television unacknowledged illegitimate child. The man fi led suit in an station once daily for one week. appropriate court alleging that the state statute barring an In which of the following ways could the city’s reward illegitimate child from sharing in a parent’s estate is invalid, offer be effectively accepted? and that he should be declared lawful heir to his father’s (A) Only by an offeree’s return promise to make a reason- estate. able effort to bring about the arrest and conviction of an In challenging the validity of the state statute, the man’s arsonist within the scope of the offer. strongest argument would be that Practice MBE—A.M. Questions 405

(A) there is no rational basis for preferring as heirs collat- (D) not prevail, because the purchaser and the man own eral relatives and even the state over unacknowledged the farm as tenants in common children, and, therefore, the law violates the Equal Protection Clause Question 7 (B) he has been deprived of property without due process A storekeeper of a large hardware store sells power saws because his fundamental right to inherit has been com- for both personal and commercial use. He often takes promised without a compelling state need old power saws as trade-ins on new ones. The old power (C) it violates the Privileges and Immunities Clause of the saws are then completely disassembled and rebuilt with Fourteenth Amendment new bearings by the storekeeper’s employees and sold by (D) it is a denial of procedural due process because it the storekeeper as ‘‘reconditioned saws.’’ A purchaser, the does not give the unacknowledged illegitimate child an owner and operator of a cabinetmaking shop, informed the opportunity to prove paternity storekeeper that he wanted to buy a reconditioned circular Question 5 saw for use in his cabinetmaking business. However, the blade that was on the saw he picked out had very coarse In which of the following situations is the defendant most teeth for cutting rough lumber. The purchaser told the store- likely to be not guilty of the charge made? keeper that he wanted a saw blade that would cut plywood. (A) The police arrested a thief and recovered goods he had The storekeeper exchanged the coarse blade for a new one stolen. At the direction of the police, the thief took the goods to the defendant. The defendant, believing the with fi ner teeth that would cut plywood smoothly. The new goods to be stolen, purchased them. The defendant is blade was manufactured by a saw-blade company and con- charged with attempting to receive stolen property. tained defects. The reconditioned saw had been manufac- (B) The defendant misrepresented his identity to secure a tured by a power-saw company. loan from a bank. The banker was not deceived and The week after the saw was purchased, the employee, refused to grant the loan. The defendant is charged who works for the purchaser in the purchaser’s cabinet- with attempting to obtain property by false pretenses. making shop, was injured while using the saw. The employ- (C) Believing that state law made it a crime to purchase ee’s arm was severely cut. As a result, the cabinetmaking codeine without a prescription, the defendant pur- shop was shut down for a week until a replacement for the chased, without a prescription, cough syrup containing employee could be found. The jurisdiction has adopted a codeine. Unknown to the defendant, the statute had been repealed and codeine could be legally purchased pure comparative fault rule in strict liability cases. without a prescription. The defendant is charged with If the employee was injured while cutting plywood when attempting to purchase codeine without a prescription. the shaft holding the saw blade came loose when a bear- (D) The defendant, intending to kill a woman, shot at the ing gave way and the shaft and blade fl ew off the saw, and woman. Unknown to the defendant, the woman had if the employee asserts a claim based on strict liability in died of a heart attack minutes before the defendant tort against the power saw company, the employee will shot at her. The defendant is charged with attempted probably murder. (A) recover if the shaft that came loose was a part of the Question 6 saw when it was new (B) recover, because the power saw company was in the A farm was conveyed to a man and a woman by a deed business of manufacturing dangerous machines that, in the jurisdiction in which the farm is situated, created (C) not recover, because the employee was not the buyer a cotenancy in equal shares and with the right of survivor- of the power saw ship. The jurisdiction has no statute directly applicable to (D) not recover, because the saw had been rebuilt by the any of the problems posed. storekeeper The woman, by deed, conveyed ‘‘my undivided one-half interest’’ in the farm to a purchaser. The woman has since Question 8 died. In an appropriate action between the purchaser and A landowner and a contractor entered into a written con- the man in which title to the farm is at issue, the man will tract under which the contractor agreed to build a building (A) prevail, because he is the sole owner of the farm and pave an adjacent sidewalk for the landowner at a price (B) prevail if, but only if, the cotenancy created in the man of $200,000. Later, while construction was proceeding, the and the woman was a tenancy by the entirety landowner and the contractor entered into an oral modifi ca- (C) not prevail if he had knowledge of the conveyance prior tion under which the contractor was not obligated to pave to the woman’s death the sidewalk, but still would be entitled to $200,000 upon 406 Strategies and Tactics for the Multistate Bar Exam

completion. The contractor completed the building. The (C) Grant judgment for the supermarket, because the landowner, after discussions with his landscaper, demanded shopper failed to offer proof of the supermarket’s that the contractor pave the adjacent sidewalk. The contrac- negligence. tor refused. (D) Submit the case to the jury, because on these facts Has the contractor breached the contract? negligence may be inferred. (A) No, because the oral modifi cation was in good faith and Question 11 therefore enforceable. The childhood home of a former U.S. president is part of a (B) Yes, because a discharge of a contractual obligation national park located in a city. The National Park Service must be in writing. entered into a contract with an independent antiques col- (C) Yes, because the parol evidence rule bars proof of the oral modifi cation. lector to acquire items owned by residents of the city dur- ing the president’s lifetime. According to the contract, the (D) Yes, because there was no consideration for the dis- charge of the contractor’s duty to pave the sidewalk. collector purchases items and then sells them to the Park Service at a price equal to the collector’s cost plus a 10 Question 9 percent commission. Purchases by antiques collectors are At a civil trial for slander, the plaintiff showed that the defen- ordinarily subject to the sales tax of the state in which the dant had called the plaintiff a thief. In defense, the defen- city is located. The collector fi les suit in state court to enjoin dant called a witness to testify, ‘‘I have been the plaintiff’s collection of the tax on these purchases, claiming that the neighbor for many years, and people in our community gen- sales tax is unconstitutional as applied to them. erally have said that he is a thief.’’ Should the state court issue the injunction? Is the testimony concerning the plaintiff’s reputation in (A) No, because as the purchaser of these antiques, the the community admissible? collector, rather than the federal government is liable for the tax. (A) No, because character is an essential element of the defense, and proof must be made by specifi c instances (B) No, because the suit is within the exclusive jurisdiction of conduct. of the federal courts. (B) Yes, to prove that the plaintiff is a thief, and to reduce or (C) Yes, because the federal government is contractually refute the damages claimed. obligated to pay the amount of the sales tax when it covers the collector’s cost of these antiques. (C) Yes, to prove that the plaintiff is a thief, but not on the issue of damages. (D) Yes, because under the Supremacy Clause, the federal program to acquire these antiques preempts the state (D) Yes, to reduce or refute the damages claimed, but not sales tax on the purchase of these items. to prove that the plaintiff is a thief. Question 12 Question 10 In a civil trial for professional malpractice, the plaintiff sought As a shopper was leaving a supermarket, an automatic to show that the defendant, an engineer, had designed the door that should have opened outward opened inward, plaintiff’s fl our mill with inadequate power. The plaintiff called striking and breaking the shopper’s nose. The owner of the an expert witness who based his testimony solely on his own building had installed the automatic door. The lease, pursu- professional experience but also asserted, when asked, that ant to which the supermarket leased the building, provided the book Smith on Milling Systems was a reliable treatise that the supermarket was responsible for all maintenance in the fi eld and consistent with his views. On cross-exam- of the premises. ination, the defendant asked the witness whether he and The shopper sued the supermarket. At trial, neither the Smith were ever wrong. The witness answered, ‘‘Nobody’s shopper nor the supermarket offered any testimony, expert perfect.’’ The defendant asked no further questions. The or otherwise, as to why the door had opened inward. At the defendant called a second expert witness and asked, ‘‘Do conclusion of the proofs, both the shopper and the super- you accept the Smith book as reliable?’’ The second wit- market moved for judgment. ness said, ‘‘It once was, but it is now badly out of date.’’ The How should the trial judge rule? plaintiff requested that the jury be allowed to examine the (A) Grant judgment for the shopper, because it is undis- book and judge for itself the book’s reliability. puted that the door malfunctioned. Should the court allow the jury to examine the book? (B) Grant judgment for the supermarket, because the shopper failed to join the owner of the building as a (A) No, because the jury may consider only passages read defendant. to it by counsel or witness. Practice MBE—A.M. Questions 407

(B) No, because the plaintiff’s expert in testifying did not a non-standard dimension for a price of $10 per set. The rely on the treatise but on his own experience. seller correctly calculated that it would cost $8 to manufac- (C) Yes, because an expert has testifi ed that the treatise is ture each set. Delivery was scheduled for 60 days later. Fifty- reliable. fi ve days later, after the seller had completed production of (D) Yes, because the jury is the judge of the weight and cred- the 1,000 sets, the buyer abandoned the project requiring ibility to be accorded both written and oral evidence. use of the specially manufactured ball bearings and repudi- ated the contract with the seller. After notifying the buyer of Question 13 his intention to resell, the seller sold the 1,000 sets of ball A car owner washed her car while it was parked on a public bearings to a salvage company for $2 per set. The seller street, in violation of a statute that prohibits the washing sued the buyer for damages. of vehicles on public streets during rush hours. The statute What damages should the court award to the seller? was enacted only to expedite the fl ow of automobile traffic. (A) $2 per set, representing the difference between the Due to a sudden and unexpected cold snap, the car own- cost of production and the price the buyer agreed to er’s waste water formed a puddle that froze. A pedestrian pay. slipped on the frozen puddle and broke her leg. The pedes- (B) $6 per set, representing the difference between the trian sued the car owner to recover for her injury. At trial, the cost of manufacture and the salvage price. only evidence the pedestrian offered as to negligence was (C) $8 per set, representing the lost profi ts plus the unre- the car owner’s admission that she had violated the statute. covered cost of production. At the conclusion of the proofs, both parties moved for a (D) Nominal damages, as the seller failed to resell the directed verdict. goods by public auction. How should the trial judge proceed? Question 16 (A) Deny both motions and submit the case to the jury, because, on the facts, the jury may infer that the car A defendant decided to kill his neighbor. He set out for his owner was negligent. neighbor’s house. Before he got there, he saw his neigh- (B) Deny both motions and submit the case to the jury, bor’s brother, who resembled the defendant’s neighbor. because the jury may consider the statutory violation Thinking the neighbor’s brother was the neighbor, the as evidence that the car owner was negligent. defendant shot at the neighbor’s brother. The shot missed (C) Grant the car owner’s motion, because the pedestrian the neighbor’s brother but wounded a bystander, who was has failed to offer adequate evidence that the car owner some distance away. The defendant had not seen the was negligent. bystander. (D) Grant the pedestrian’s motion, because of the car own- In a prosecution under a statute that proscribes attempt er’s admitted statutory violation. to commit murder, the district attorney should indicate that Question 14 the intended victim(s) was (were) A dentist was anesthetizing a patient’s gum before pulling a (A) The neighbor only tooth. Although the dentist used due care, the hypodermic (B) The neighbor’s brother only needle broke off in the patient’s gum, causing injury. The (C) The bystander only needle broke because of a manufacturing defect that the (D) The neighbor and the neighbor’s brother dentist could not have detected. Question 17 Is the patient likely to recover damages in an action against the dentist based on strict products liability and At the defendant’s trial for a gang-related murder, the pros- malpractice? ecution introduced, as former testimony, a statement by a gang member who testifi ed against the defendant at a pre- (A) No, on neither basis. liminary hearing and has now invoked his privilege against (B) Yes, based on malpractice, but not on strict products liability. self-incrimination. (C) Yes, based on strict products liability, but not on If the defendant now seeks to impeach the credibility of malpractice. the gang member, which of the following is the court most (D) Yes, on both bases. likely to admit? (A) Evidence that the gang member had three misde- Question 15 meanor convictions for assault. A buyer entered into a written contract to purchase from a (B) Testimony by a psychologist that persons with the gang seller 1,000 sets of specially manufactured ball bearings of member’s background have a tendency to fabricate. 408 Strategies and Tactics for the Multistate Bar Exam

(C) Testimony by a witness that at the time the gang mem- (C) No, because the purchase at the foreclosure sale by ber testifi ed, he was challenging the defendant’s lead- the buyer under these facts eliminated the second ership role in the gang. bank’s junior mortgage lien. (D) Testimony by a witness that the gang member is a (D) No, because of the due-on-sale provision in the farm- cocaine dealer. er’s mortgage to the fi rst bank.

Question 18 Question 20 While negligently driving his father’s uninsured automobile, A woman offered to pay her friend one-third of the stolen a 25-year-old student crashed into an automobile driven by proceeds if the friend would drive the getaway car to be a woman. Both the student and the woman were injured. used in a bank robbery. The friend agreed but made the The student’s father, erroneously believing that he was lia- woman promise not to hurt anyone during . ble because he owned the automobile, said to the woman: ‘‘I The woman then drove to a sporting goods store, will see to it that you are reimbursed for any losses you incur where she explained to the store owner that she needed as a result of the accident.’’ a small fi rearm for use in a bank robbery. The store owner In an action by the woman against the student’s father responded that he would charge extra because the woman for wages lost while she was incapacitated as a result of the was so unwise as to confi de her unlawful plans for using accident, which of the following would be the father’s best the weapon, and he sold her a handgun at four times the defense? regular price. (A) Lack of consideration During the robbery, the woman used the gun to (B) Mistake of fact as to basic assumption threaten a bank teller into handing over . The gun (C) Statute of Frauds discharged by accident and killed a bank customer. (D) Indefi niteness of father’s promise At common law, who in addition to the woman could properly be convicted of murder in the death of the Question 19 customer? A farmer borrowed $100,000 from a bank and gave the bank (A) Both the friend and the store owner. a promissory note secured by a mortgage on the farm that (B) Neither the friend nor the store owner. she owned. The bank promptly and properly recorded the (C) Only the friend. mortgage, which contained a due-on-sale provision. (D) Only the store owner. A few years later, the farmer borrowed $5,000 from a second bank and gave it a promissory note secured by Question 21 a mortgage on her farm. The bank promptly and properly On May 1, an uncle mailed a letter to his adult nephew that recorded the mortgage. stated: ‘‘I am thinking of selling my pickup truck, which you Subsequently, the farmer defaulted on her obligation have seen and ridden in. I would consider taking $7,000 for to the fi rst bank, which then validly accelerated the debt it.’’ On May 3, the nephew mailed the following response: ‘‘I and instituted nonjudicial foreclosure proceedings as per- will buy your pickup for $7,000 cash.’’ The uncle received this mitted by the jurisdiction. The second bank received notice letter on May 5 and on May 6 mailed a note that stated: ‘‘It’s of the foreclosure sale but did not send a representative to a deal.’’ On May 7, before the nephew had received the sale. At the foreclosure sale, a buyer who was not act- of May 6, he phoned his uncle to report that he no longer ing in collusion with the farmer outbid all other bidders and wanted to buy the pickup truck because his driver’s license received a deed to the farm. had been suspended. Several months later, the original farmer repurchased Which of the following statements concerning this her farm from the buyer, who executed a warranty deed exchange is accurate? transferring the farm to her. After the farmer promptly and (A) There is a contract as of May 3. properly recorded that deed, the second bank commenced (B) There is a contract as of May 5. foreclosure proceedings on the farm. The farmer denied the (C) There is a contract as of May 6. validity of the second bank’s mortgage. (D) There is no contract. Does the second bank continue to have a valid mort- gage on the farm? Question 22 (A) Yes, because of the doctrine of estoppel by deed. In a civil action for misrepresentation in the sale of real (B) Yes, because the original owner reacquired title to the estate, the parties contested whether the defendant was farm. licensed by the State Board of Realtors, a public agency Practice MBE—A.M. Questions 409 established by statute to license real estate brokers. The The client has fi led suit against both lawyers for negli- defendant testifi ed she was licensed. On rebuttal, the gence. That case is on trial with a jury in a court of general plaintiff offers a certifi cation, bearing the seal of the secre- jurisdiction. tary of the State Board of Realtors. The certifi cation states In order to establish a breach of standard of care owed that conducted a thorough search of the to her by the two lawyers, the client agency’s records and all relevant databases, and that this (A) must have a legal expert from the same locality testify search uncovered no record of a license ever having been that the defendants’ conduct was a breach issued to the defendant. The certifi cation is signed by the (B) must have a legal expert from the same state testify secretary. that the defendants’ conduct was a breach Is the certifi cation that there was no record of a license (C) can rely on the application of the jurors’ common knowl- issuance admissible? edge as to whether there was a breach (A) No, because it is hearsay not within any exception. (D) can rely on the judge, as an expert in the law, to advise the jury whether there was a breach (B) No, because the writing was not properly authenticated. Question 25 (C) Yes, for the limited purpose of impeaching the defendant. A plaintiff sued her employer, alleging that poor working conditions had caused her to develop a stomach ulcer. At (D) Yes, to prove the nonexistence of a public record. trial, the plaintiff’s medical expert testifi ed to the cause of Question 23 the plaintiff’s ulcer and stated that his opinion was based in part on information in a letter the plaintiff’s personal physi- A federal statute imposes an excise tax of $100 on each cian had written to the plaintiff’s employer, explaining why new computer sold in the United States. It also appropri- the plaintiff had missed work. ates the entire proceeds of that tax to a special fund, which When offered to prove the cause of the plaintiff’s condi- is required to be used to purchase licenses for computer tion, is the letter from the plaintiff’s doctor admissible? software that will be made available for use, free of charge, to any resident of the United States. (A) No, because it is hearsay not within any exception. Is this statute constitutional? (B) No, because the plaintiff’s physician is not shown to be unavailable. (A) No, because the federal government may not impose any direct taxes on citizens of the United States. (C) Yes, because it was relied upon by the plaintiff’s medi- cal expert. (B) No, because this statute takes without just compensa- tion the property of persons who hold patents or copy- (D) Yes, under the business records exception to the hear- rights on computer software. say rule. (C) Yes, because it is a reasonable exercise of the power of Question 26 Congress to tax and spend for the general welfare. A customer wanted to purchase a used motor vehicle. The (D) Yes, because the patent power authorizes Congress to impose reasonable charges on the sale of technology used car company’s lot, in a remote section away from town, and to spend the proceeds of those charges to advance was enclosed by a ten-foot chain link fence. While the cus- the use of technology in the United States. tomer and a sales representative, an employee of the used car company, were in the used car lot looking at cars, a Question 24 security guard locked the gate at 1:30 p.m., because it was Two lawyers are the only members of a law partnership in Saturday and the lot was supposed to be closed after 1:00 a small town that has only one other lawyer in it. Both attor- p.m. Saturday until Monday morning. At 1:45 p.m., the cus- neys do a substantial amount of personal injury work. A cli- tomer and the sales representative discovered they were ent was severely and permanently injured in an automobile locked in. collision when struck by an automobile driven by a motorist. There was no traffic in the vicinity and no way in which The client employed the law partnership to represent her help could be summoned. After two hours, the customer in obtaining damages for her injuries. At the time the client began to panic at the prospect of remaining undiscovered employed the partnership, the statute of limitations had six and without food and water until Monday morning. The sales weeks to run on her claim. The complaint was prepared but representative decided to wait in a car until help came. The not fi led. Each lawyer thought that the other lawyer would customer tried to climb over the fence and, in doing so, fell, fi le the complaint. The statute of limitations ran out on the and was injured. The customer asserts a claim against the client’s claim against the motorist. used car company for damages for his injuries. 410 Strategies and Tactics for the Multistate Bar Exam

If the customer’s claim is based on false imprisonment, Last year, a major new medical center was constructed will the customer prevail? adjacent to the subdivision. A doctor who owns a house in (A) Yes, because he was confi ned against his will. the subdivision wishes to relocate her medical offices to her (B) Yes, because he was harmed as a result of his house. For the fi rst time, the doctor learned of the restrictive confi nement. covenant in the deed from the man to the developer. The (C) No, unless the security guard was negligent in locking applicable zoning ordinance permits the doctor’s intended the gate. use. The man, as owner of the fi ve-acre tract, however, (D) No, unless the security guard knew that someone was objects to the doctor’s proposed use of her property. in the lot at the time the guard locked the gate. There are no governing statutes other than the zoning code. The common-law Rule Against Perpetuities is unmod- Question 27 ifi ed in the jurisdiction. An investor offered a landowner $200 for a 30-day option Can the doctor convert her house in the subdivision to buy the landowner’s land for $10,000. As the landowner into a medical office? knew, the investor, if granted the option, intended to resell (A) No, because the owners of lots in the subdivision own the land at a profi t. The landowner declined, believing that property benefi tted by the original residential covenant she could fi nd a desirable purchaser herself. The investor and have the sole right to enforce it. thereupon said to the landowner, ‘‘Make me a written, 30-day (B) No, because the man owns property benefi tted by offer, revocable at your pleasure, to sell me your land at a the original restrictive covenant and has a right to sale price of $10,000, and tomorrow I will pay you $200 for enforce it. so doing.’’ The landowner agreed and gave the investor the (C) Yes, because the original restrictive covenant violates following signed document: the Rule Against Perpetuities. For 30 days I offer my land to the investor for $10,000, (D) Yes, because the zoning ordinance allows the doctor’s this offer to be revocable at my pleasure at any time before proposed use and preempts the restrictive covenant. acceptance. Which of the following would best describe the basis of Question 29 any duty or duties created by the investor’s oral promise and With the advice and consent of the Senate, the President the landowner’s writing? entered into a self-executing treaty with a foreign coun- (A) Firm option try. The treaty provided that citizens of both nations were required to pay whatever torts damages were awarded (B) Precontractual liability by promissory estoppel against them by a court of either nation. (C) Unilateral contract A man and a woman who were U.S. citizens and res- (D) Quasi-contractual liability idents of the same state were traveling separately in the Question 28 foreign country when their cars collided. The foreign court Twenty-fi ve years ago, a man who owned a 45-acre tract awarded the woman a judgment for $500,000 in damages of land conveyed 40 of the 45 acres to a developer by war- for her injuries from the accident. ranty deed. The man retained the rear fi ve-acre portion of In federal district court in their home state, the woman the land and continues to live there in a large farmhouse. fi led suit against the man to enforce the judgment. The man The deed to the 40-acre tract was promptly and prop- fi led a motion to dismiss for lack of jurisdiction. erly recorded. It contained the following language: Should the court grant the motion to dismiss? It is a term and condition of this deed, which shall be a (A) Yes, because the citizenship of the parties is not covenant running with the land and binding on all owners, diverse. their heirs and assigns, that no use shall be made of the (B) Yes, because the traffic accident was a noncommercial 40-acre tract of land except for residential purposes. transaction outside interstate commerce. Subsequently, the developer fully developed the (C) No, because the case falls within the federal question 40-acre tract into a residential subdivision consisting of 40 jurisdiction of the court. lots with a single-family residence on each lot. (D) No, because the treaty power is plenary and not sub- ject to judicial review. Although there have been multiple transfers of owner- ship of each of the 40 lots within the subdivision, none of Question 30 them included a reference to the quoted provision in the Section 1 of the Vehicle Code of a state makes it illegal to deed from the man to the developer, nor did any deed to a cross a street in a central business district other than at a subdivision lot create any new covenants restricting use. Practice MBE—A.M. Questions 411 designated crosswalk. Section 2 of the Code prohibits park- kidnapped an American citizen wanted in the United States ing any motor vehicle so that it blocks any part of a desig- for drug smuggling violations, and forcibly drove him back nated crosswalk. to Texas. Thereafter, the agents, again without a warrant, A pedestrian wanted to cross Main Street in the central broke into the Texas home of the accomplice of the kid- business district of a city, located in the state at issue, but a napped citizen, and arrested her. truck parked by a trucker was blocking the designated cross- The kidnapped citizen and his accomplice were both walk. The pedestrian stepped out into Main Street and care- indicted for narcotics violations. Both moved to dismiss fully walked around the back of the truck. The pedestrian was the indictment on the ground that their arrests violated the struck by a motor vehicle negligently operated by a driver. Fourth Amendment. If the pedestrian asserts a claim against the driver, the The court should pedestrian’s failure to be in the crosswalk will have which of (A) grant the motions of both the kidnapped citizen and his the following effects? accomplice. (A) It is not relevant in determining the right of the (B) grant the motion of the kidnapped citizen and deny the pedestrian. motion of his accomplice. (B) It may be considered by the trier of facts on the issue of (C) grant the motion of the accomplice and deny the motion the driver’s liability. of the kidnapped citizen. (C) It will bar the pedestrian’s recovery unless the driver (D) deny the motions of both the kidnapped citizen and his saw the pedestrian in time to avoid the impact. accomplice. (D) It will bar the pedestrian’s recovery as a matter of law. Question 33 Question 31 A victim was held up at the point of a gun, an unusual An innkeeper, who had no previous experience in the motel revolver with a red-painted barrel, while she was clerking or commercial laundry business and who knew nothing in a neighborhood grocery store. The defendant is charged about the trade usages of either business, bought a motel with armed robbery of the victim. and signed an agreement with a laundry company for the The prosecutor calls a witness to testify that, a week motel’s laundry services. The one-year agreement provided after the robbery of the victim, he was robbed by the defen- for ‘‘daily service at $500 a week.’’ From their conversations dant with a pistol that had red paint on the barrel. The wit- during negotiation, the laundry company knew that the ness’s testimony is innkeeper expected laundry services seven days a week. (A) admissible as establishing an identifying circumstance When the laundry company refused to pick up the motel’s (B) admissible as showing that the defendant was willing to laundry on two successive Sundays and indicated that it commit robbery would not ever do so, the innkeeper canceled the agree- (C) inadmissible, because it is improper character evi- ment. The laundry company sued the innkeeper for breach dence of contract. At trial, clear evidence was introduced to show (D) inadmissible, because its probative value is substan- that in the commercial laundry business ‘‘daily service’’ did tially outweighed by the danger of unfair prejudice not include service on Sundays. Will the laundry company succeed in its action? Question 34 (A) No, because the laundry company knew the meaning An act of Congress provides that ‘‘no federal court shall the innkeeper attached to ‘‘daily service,’’ and, there- order the implementation of a public school desegregation fore, the innkeeper’s meaning will control. plan that would require the transportation of any student to (B) No, because the parties attached materially different a school other than the school closest or next closest to his meanings to ‘‘daily service,’’ and, therefore, no contract place of residence.’’ was formed. Which of the following is the strongest argument for the (C) Yes, because the parol evidence rule will not permit the constitutionality of the act? innkeeper to prove the meaning she attached to ‘‘daily service.’’ (A) The Fourteenth Amendment authorizes Congress to defi ne governmental conduct that violates the Equal (D) Yes, because the trade usage will control the interpre- Protection Clause. tation of ‘‘daily service.’’ (B) Under Article III, Congress may restrict the jurisdiction Question 32 of the federal courts. FBI agents, without a warrant and without permission of (C) Transportation of students is subject to regulation by Mexican law enforcement or judicial officers, entered Mexico, Congress because commerce is involved. 412 Strategies and Tactics for the Multistate Bar Exam

(D) Congress provides partial support for public education (D) The written contract controls, and the buyer is entitled and is therefore entitled to establish conditions upon to no discount because of the parol evidence rule. the expenditure of federal grants. Question 37 Question 35 In which of the following situations is the defendant most A hiker sustained a head injury when he was struck by a likely to be guilty of common-law murder? limb that fell from a tree. At the time of his injury, the hiker (A) Angered because his neighbor is having a noisy party, was walking through a forest on private property without the the defendant fi res a rifl e into the neighbor’s house. property owner’s knowledge or permission. It was deter- The bullet strikes and kills a guest at the party. mined that the limb fell because the tree was infested with (B) During an argument, the defendant’s cousin slaps the termites. defendant. Angered, the defendant responds by shoot- In an action by the hiker against the property owner to ing and killing his cousin. recover for his head injury, will the hiker prevail? (C) The defendant drives his car through a red light and strikes and kills a pedestrian who is crossing the (A) No, because the property owner could not foresee that street. anyone would be injured. (D) Using his fi st, the defendant punches a victim in the (B) No, because the property owner breached no duty to face. As a result of the blow, the victim falls and hits the hiker, who was a trespasser. his head on a concrete curb, suffers a concussion, and (C) Yes, because the property owner had a duty to prevent dies. the trees on his property from becoming dangerous. (D) Yes, because the property owner is liable for hidden Question 38 dangers on his property. A defendant is tried for armed robbery of a bank. The pro- secution, in its case in chief, offers evidence that when the Question 36 defendant was arrested one day after the crime, he had a A seller and a buyer have dealt with each other in hun- quantity of heroin and a hypodermic needle in his dreds of separate grain contracts over the last fi ve years. possession. In performing each contract, the seller delivered the grain This evidence should be to the buyer and, upon delivery, the buyer signed an invoice (A) admitted to prove the defendant’s motive to commit the that showed an agreed-upon price for that delivery. Each crime invoice was silent in regard to any discount from the price (B) admitted to prove the defendant’s propensity to commit in exchange for prompt payment. The custom of the grain crimes trade is to allow a 2 percent discount from the invoice price (C) excluded, because its probative value is substantially for payment within ten days of delivery. In all of their prior outweighed by the danger of unfair prejudice transactions and without objection from the seller, the buyer (D) excluded, because such evidence may be offered only took 15 days to pay and deducted 5 percent from the invoice to rebut evidence of good character offered by the price. The same delivery procedure and invoice were used defendant in the present contract as had been used previously. The Question 39 present contract called for a single delivery of wheat at a A man contacted his lawyer regarding his right to use a path price of $300,000. The seller delivered the wheat and the on his neighbor’s vacant land. buyer then signed the invoice. On the third day after deliv- Fifteen years ago, after a part of the path located on his ery, the buyer received the following note from the seller: land and connecting his cabin to the public highway washed ‘‘Payment in full in accordance with signed invoice is due out, the man cleared a small part of his neighbor’s land and immediately. No discounts permitted.’’ s/Seller. rerouted a section of the path through the neighbor’s land. Which of the following statements concerning these Twelve years ago, the neighbor leased her land to facts is most accurate? some hunters. For the next 12 years, the hunters and the (A) The custom of the trade controls, and the buyer is entitled man who had rerouted the path used the path for access to take a 2 percent discount if he pays within ten days. to the highway. (B) The parties’ course of dealing controls, and the buyer A month ago, the neighbor discovered that part of the is entitled to take a 5 percent discount if he pays within 15 days. path was on her land. The neighbor told the man that she (C) The seller’s retraction of his prior waiver controls, and had not given him permission to cross her land and that she the buyer is entitled to no discount. would be closing the rerouted path after 90 days. Practice MBE—A.M. Questions 413

The man’s land and the neighbor’s land have never (B) Uphold the law, because the legislature could rationally been in common ownership. believe that diagnostic centers not affiliated with hospi- The period of time necessary to acquire rights by pre- tals would be less reliable than hospitals. scription in the jurisdiction is ten years. The period of time (C) Invalidate the law, because it imposes an undue bur- necessary to acquire title by adverse possession in the den on access to medical services in the state. jurisdiction is ten years. (D) Dismiss the suit without reaching the merits, because the suit is not ripe. What should the lawyer tell the man concerning his right to use the rerouted path on the neighbor’s land? Question 42 (A) The man has fee title by adverse possession of the Until 1954, a state required segregation in all public and pri- land included in the path. vate schools, but all public schools are now desegregated. (B) The man has an easement by necessity to use the Other state laws, enacted before 1954 and continuing to the path. present, provide for free distribution of the same textbooks (C) The man has an easement by prescription to use the path. on secular subjects to students in all public and private schools. In addition, the state accredits schools and certi- (D) The man has no right to use the path. fi es teachers. Question 40 A private school that offers elementary and second- A debtor owed a lender $1,500. The statute of limitations ary education in the state, denies admission to all non- barred recovery on the claim. The debtor wrote to the lender, Caucasians. stating, ‘‘I promise to pay you $500 if you will extinguish the Which of the following is the strongest argument against debt.’’ The lender agreed. the constitutionality of free distribution of textbooks to the Is the debtor’s promise to pay the lender $500 students at the private school? enforceable? (A) No legitimate educational function is served by the free distribution of the textbooks. (A) No, because the debtor made no promise not to plead the statute of limitations as a defense. (B) The state may not in any way aid private schools. (B) No, because there was no consideration for the debt- (C) The Constitution forbids private bias of any kind. or’s promise. (D) Segregation is furthered by the distribution of textbooks (C) Yes, because the debtor’s promise provided a benefi t to to these students. the lender. Question 43 (D) Yes, because the debtor’s promise to pay part of the barred antecedent debt is enforceable. In a civil trial for fraud arising from a real estate transac- tion, the defendant claimed not to have been involved in the Question 41 transaction. The plaintiff called a witness to testify concern- In one state, certain kinds of advanced diagnostic medical ing the defendant’s involvement in the fraudulent scheme, technology were located only in hospitals, where they pro- but to the plaintiff’s surprise the witness testifi ed that the vided a major source of revenue. In many other states, such defendant was not involved, and denied making any state- technology was also available at ‘‘diagnostic centers’’ that ment to the contrary. The plaintiff now calls a second wit- were not affiliated with hospitals. ness to testify that the fi rst witness had stated, while the two A group of physicians announced its plan to immedi- were having a dinner conversation, that the defendant was ately open in the state a diagnostic center that would not involved in the fraudulent transaction. be affiliated with a hospital. The state hospital association Is the testimony of the second witness admissible? argued to the state legislature that only hospitals could reli- (A) No, because a party cannot impeach the party’s own ably handle advanced medical technologies. The legislature witness. then enacted a law prohibiting the operation in the state of (B) No, because it is hearsay not within any exception. diagnostic centers that were not affiliated with hospitals. (C) Yes, but only to impeach the fi rst witness. The group of physicians fi led suit challenging the con- (D) Yes, to impeach the fi rst witness and to prove the defen- stitutionality of the state law. dant’s involvement. What action should the court take? Question 44 (A) Uphold the law, because the provision of medical ser- vices is traditionally a matter of legitimate local concern By a valid written contract, a seller agreed to sell land to a that states have unreviewable authority to regulate. buyer. The contract stated, ‘‘The parties agree that closing 414 Strategies and Tactics for the Multistate Bar Exam

will occur on next May 1 at 10 a.m.’’ There was no other sidewalk in order to enjoy the feel of walking or running on reference to closing. The contract was silent as to quality grass. He put up a ‘‘No Trespassing’’ sign, but it did not stop of title. the practice. He then put up a ‘‘Beware of Skunk’’ sign and On April 27, the seller notifi ed the buyer that she had bought a young skunk. He took the skunk to a vet to have discovered that the land was subject to a longstanding its scent gland removed. Unfortunately, the vet did not per- easement in favor of a corporation for a towpath for a canal, form the operation properly, and the scent gland was not should the corporation ever want to build a canal. removed. The homeowner was unaware that it had not been The buyer thought it so unlikely that a canal would be removed. built that the closing should occur notwithstanding this out- One day a walker was out for a stroll. When she came to standing easement. Therefore, the buyer notifi ed the seller the homeowner’s property, she walked on the grass along- on April 28 that he would expect to close on May 1. side the sidewalk on the homeowner’s property. The skunk When the seller refused to close, the buyer sued for came up behind the walker and sprayed her with its scent. specifi c performance. The smell was overpowering, and she fainted. She struck Will the buyer prevail? her head on the sidewalk and suffered serious injuries. (A) No, because the easement renders the seller’s title The probable result of the walker’s claim against the unmarketable. homeowner is that she will (B) No, because rights of third parties are unresolved. (A) recover, because the skunk was a private nuisance (C) Yes, because the decision to terminate the contract for (B) recover, because the skunk was not a domesticated title not being marketable belongs only to the buyer. animal (D) Yes, because the seller did not give notice of the ease- (C) not recover, because the walker was a trespasser ment a reasonable time before the closing date. (D) not recover, because the vet was the cause of the injury Question 45 In a civil action, the plaintiff sued a decedent’s estate to Question 47 recover damages for the injuries she suffered in a colli- A woman told a man to go into her friend’s unlocked barn sion between her car and one driven by the decedent. At and retrieve an expensive black saddle that she said she trial, the plaintiff introduced undisputed evidence that the had loaned to the friend. The man went to the friend’s barn, decedent’s car swerved across the median of the highway, opened the door, found a black saddle, and took it back to where it collided with an oncoming car driven by the plaintiff. the woman’s house. The friend had in fact not borrowed a The decedent’s estate introduced undisputed evidence that, saddle from the woman, and when the friend discovered prior to the car’s crossing the median, the decedent suffered her black saddle missing, she suspected that the woman a fatal heart attack, which she had no reason to foresee, was the thief. The friend used a screwdriver to break into and that, prior to the heart attack, the decedent had been the woman’s house to fi nd the saddle. Upon discovering the driving at a reasonable speed and in a reasonable manner. saddle on the woman’s table, the friend took it back and A statute makes it a traffic offense to cross the median of a called the police. highway. The jurisdiction follows the common law, except that In this case, for whom should the court render burglary covers structures in addition to dwellings and the judgment? nighttime element has been eliminated. (A) The decedent’s estate, because its evidence is Which, if any, of these individuals is guilty of burglary? undisputed. (A) All of them. (B) The decedent’s estate, because the plaintiff has not (B) Only the friend. established a prima facie case of liability. (C) Only the man. (C) The plaintiff, because the accident was of a type that does not ordinarily happen in the absence of negli- (D) Only the woman. gence on the actor’s part. Question 48 (D) The plaintiff, because the decedent crossed the median in violation of the statute. A mathematician was hired as an assistant professor of mathematics at a state college and is now in his third Question 46 consecutive one-year contract. Under state law he can- A homeowner resented the fact that joggers and walkers not acquire tenure until after fi ve consecutive annual con- would sometimes come onto his property just beside the tracts. In his third year, the mathematician was notifi ed that Practice MBE—A.M. Questions 415 he was not being rehired for the following year. Applicable (B) sustained, because the defendant was not represented state law and college rules did not require either a state- by counsel at the showing of the photographs to the ment of reasons or a hearing, and in fact neither was victim offered to him. (C) sustained, because the action of the police in show- Which of the following, if established, sets forth the ing the photographs to the victim was unnecessarily suggestive strongest constitutional argument the mathematician could (D) denied make to compel the college to furnish him a statement of reasons for the failure to rehire him and an opportunity for Question 51 a hearing? On May 1, a seller and a buyer entered into a written con- (A) There is no evidence that tenured teachers are any tract, signed by both parties, for the sale of a tract of land more qualifi ed than he is. for $100,000. Delivery of the deed and payment of the pur- (B) He leased a home in reliance on an oral promise of chase price were scheduled for July 1. On June 1, the buyer reemployment by the college president. received a letter from the seller repudiating the contract. On (C) He was the only teacher at the college whose contract June 5, the buyer bought a second tract of land at a higher was not renewed that year. price as a substitute for the fi rst tract. On June 10, the seller (D) In the expectation of remaining at the college, he had just moved his elderly parents to the town in which the communicated a retraction of the repudiation to the buyer. college is located. The buyer did not tender the purchase price for the fi rst tract on July 1, but subsequently sued the seller for breach Question 49 of contract. An insurance company issued an insurance policy to a Will the buyer likely prevail? homeowner. The policy failed to contain certain coverage (A) No, because the seller retracted the repudiation prior to terms required by a state insurance statute. When the the agreed time for performance. homeowner suffered a loss due to a theft that was within (B) No, because the buyer’s tender of the purchase price the policy’s terms, the insurance company refused to pay, on July 1 was a constructive condition to the seller’s claiming that the contract was unenforceable because it duty to tender a conveyance. violated the statute. (C) Yes, because the seller’s repudiation was non-retract- Will the homeowner succeed in an action against the able after it was communicated to the buyer. insurance company to recover for the loss? (D) Yes, because the buyer bought the second tract as a substitute for the fi rst tract prior to the seller’s (A) No, because the insurance policy is not a divisible retraction. contract. (B) No, because the insurance policy violated the statute. Question 52 (C) Yes, because the homeowner belongs to the class of On a parcel of land immediately adjacent to a woman’s persons intended to be protected by the statute. 50-acre farm, a public school district built a large consoli- (D) Yes, because the insurance policy would be strictly con- dated high school that included a 5,000-seat lighted athletic strued against the insurance company as the drafter. stadium. The woman had objected to the district’s plans for Question 50 the stadium and was particularly upset about nighttime ath- On a foggy night, a victim was clubbed from behind by a letic events that attracted large crowds and that, at times, man wielding a blackjack. The defendant was arrested in resulted in signifi cant noise and light intensity levels. On the vicinity shortly thereafter. As they were booking the nights of athletic events, the woman and her family mem- defendant, the police took his photograph. They promptly bers wore earplugs and could not sleep or enjoy a quiet showed that photograph, along with the photographs of evening until after 10 p.m. In addition, light from the stadium seven people who had the same general features as the on those nights was bright enough to allow reading a news- defendant, to the victim. The victim identifi ed the defendant paper in the woman’s yard. as the culprit. Which of the following doctrines would best support the At trial, the defendant objects to the introduction into woman’s claim for damages? evidence of his out-of-court identifi cation. His objection (A) Constructive eviction should be (B) Private nuisance (A) sustained, because the victim did not have a good (C) Public nuisance opportunity to observe the culprit (D) Waste 416 Strategies and Tactics for the Multistate Bar Exam

Question 53 Question 55 On March 1, an excavator entered into a contract with a The owner in fee simple of a small farm consisting of 30 contractor to perform excavation work on a large project. acres of land improved with a house and several outbuild- The contract expressly required that the excavator begin ings, leased the same to a farmer for a ten-year period. work on June 1 to enable other subcontractors to install After two years had expired, the government condemned utilities. On May 15, the excavator requested a 30-day delay 20 acres of the property and allocated the compensa- in the start date for the excavation work because he was tion award to the owner and the farmer according to their seriously behind schedule on another project. When the respective interest so taken. It so happened, however, that contractor refused to grant the delay, the excavator stated the 20 acres taken embraced all of the farm’s tillable land, that he would try to begin the work for the contractor on leaving only the house, outbuildings, and a small wooded June 1. lot. There is no applicable statute in the jurisdiction where Does the contractor have valid legal grounds to cancel the property is located nor any provision in the lease relat- the contract with the excavator and hire a replacement? ing to condemnation. The farmer quit possession, and the (A) Yes, because the excavator committed an anticipatory owner brought suit against him to recover rent. The owner repudiation of the contract by causing the contractor to will feel insecure about the performance. (A) lose, because there has been a frustration of purpose (B) Yes, because the excavator breached the implied cov- that excuses the farmer from further performance of his enant of good faith and fair dealing. contract to pay rent (C) No, because the excavator would be entitled to spe- (B) lose, because there has been a breach of the implied cifi c performance of the contract if he could begin by covenant of quiet enjoyment by the owner’s inability to June 1. provide the farmer with possession of the whole of the (D) No, because the excavator did not state unequivocally property for the entire term that he would delay the beginning of his work. (C) win, because of the implied warranty on the part of the tenant to return the demised premises in the same Question 54 condition at the end of the term as they were at the Ten years ago, a labor leader divorced his wife. Both beginning he and his fi rst wife have since married other persons. (D) win, because the relationship of landlord and tenant Recently, a newspaper in another city ran a feature article was unaffected by the condemnation, thus leaving the on improper infl uences it asserted had been used by labor farmer still obligated to pay rent officials to secure favorable rulings from government offi- Question 56 cials. The story said that in 1980 the labor leader’s fi rst Statutes in the jurisdiction defi ne criminal assault as ‘‘an wife, with his knowledge and concurrence, gave sexual attempt to commit a criminal battery’’ and criminal battery favors to the mayor of the labor leader’s hometown and as ‘‘causing an offensive touching.’’ then persuaded the mayor to grant concessions to the As a defendant was walking down the street, a gust of labor leader’s union. wind blew off his hat. The defendant reached out, trying to The story named the labor leader and identifi ed his fi rst grab his hat, and narrowly missed striking the plaintiff in by her former and current surnames. The reporter for face with his hand. the newspaper believed the story to be true, since it had If charged with criminal assault, the defendant should been related to him by two very reliable sources. be found The labor leader’s fi rst wife suffered emotional distress and became very depressed. If she asserts a claim based (A) guilty, because he caused the plaintiff to be in appre- hension of an offensive touching on defamation against the newspaper, she will (B) guilty, because he should have realized he might strike (A) prevail, because the story concerned her personal, pri- someone by reaching out vate life (C) not guilty, because he did not intend to hit the plaintiff (B) prevail if the story was false (D) not guilty, because he did not hit the plaintiff (C) not prevail, because the newspaper did not print the story with knowledge of its falsity or with reckless dis- Question 57 regard for its truth or falsity Residents of a city complained that brightly colored signs (D) not prevail if the newspaper exercised ordinary care in determining if the story was true or false detracted from the character of the city’s historic district and Practice MBE—A.M. Questions 417 distracted motorists trying to navigate its narrow streets. In specifi cations on the lot and to sell the house and lot to response, the city council enacted an ordinance requiring the contractor. The contract provided for an ‘‘inside date’’ any ‘‘sign or visual display’’ visible on the streets of the his- of April 1, 2010, and an ‘‘outside date’’ of May 1, 2010, for toric district to be black and white and to be no more than completion of the house and delivery of a deed. Neither four feet long or wide. party tendered performance on the dates stated. On May 3, A political party wanted to hang a six-foot-long, red, 2010, the contractor notifi ed the landowner in writing of the white, and blue political banner in front of a building in the contractor’s election to cancel the contract because of the historic district. The party fi led suit to challenge the consti- landowner’s failure to deliver title by May 1. On May 12, the tutionality of the sign ordinance as applied to the display of landowner notifi ed the contractor that some unanticipated its banner. construction difficulties had been encountered but that the Which of the following would be the most useful argu- landowner was entitled to a reasonable time to complete in ment for the political party? any event. The notifi cation also included a promise that the (A) The ordinance is not the least restrictive means of pro- landowner would be ready to perform by May 29 and that moting a compelling government interest. he was setting that date as an adjourned closing date. The (B) The ordinance is not narrowly tailored to an important landowner obtained a certifi cate of occupancy and appro- government interest, nor does it leave open alternative priate documents of title, and he tendered performance on channels of communication. May 29. The contractor refused. The landowner brought an (C) The ordinance imposes a prior restraint on political action to recover damages for breach of contract. The deci- expression. sion in the case will most likely be determined by whether (D) The ordinance effectively favors some categories of (A) the landowner acted with due diligence in completing speech over others. the house (B) the contractor can prove actual ‘‘undue hardship’’ Question 58 caused by the delay A car dealer owed a bank $10,000, due on June 1. The car (C) the expressions ‘‘inside date’’ and ‘‘outside date’’ are dealer subsequently sold an automobile to a buyer at a construed to make time of the essence price of $10,000, payable at $1,000 per month beginning (D) there is a showing of good faith in the contractor’s on June 1. The car dealer then asked the bank whether efforts to terminate the contract the bank would accept payments of $1,000 per month for Question 60 ten months beginning June 1, without interest, in payment of the debt. The bank agreed to that arrangement and the A man owned a much-loved cat, worth about $25, that fre- car dealer then directed the buyer to make the payments quently trespassed on a neighbor’s property. The neighbor to the bank. When the buyer tendered the fi rst payment to repeatedly asked the man to keep the cat on his own prop- the bank, the bank refused the payment, asserting that it erty, but the trespasses did not diminish. Aware of the man’s would accept payment only from the car dealer. On June 2, attachment to the cat, the neighbor killed the cat with a shot- the bank demanded that the car dealer pay the debt in full gun in full view of the man. As a consequence, the man immediately. The car dealer refused to pay and the bank suffered great emotional distress. sued the car dealer to recover the $10,000. In an action by the man against the neighbor, which of In this suit, which of the following arguments best sup- the following claims would be likely to result in the greatest ports the bank’s claim for immediate payment? monetary recovery? (A) The agreement to extend the time for payment was not (A) Battery in writing. (B) Intentional infl iction of mental suffering (B) The car dealer could not delegate its duty to pay to the (C) Trespass to chattel buyer. (D) Conversion (C) The car dealer gave no consideration for the agree- ment to extend the time of payment. Question 61 (D) The car dealer’s conduct was an attempted novation A plaintiff sued a defendant for injuries allegedly suffered that the bank could reject. when he slipped and fell on the defendant’s business prop- erty. Without asking that the defendant’s property manager Question 59 be declared a hostile witness, the plaintiff called him solely A landowner owned a vacant lot. He entered into a writ- to establish that the defendant was the owner of the prop- ten contract with a contractor to build a house of stated erty where the plaintiff fell. On cross-examination of the 418 Strategies and Tactics for the Multistate Bar Exam

manager, the defendant’s attorney sought to establish that landowner handed the instrument to the friend, saying, the defendant had taken reasonable precautions to make ‘‘This is yours, but please do not record it until after I am the property safe for business invitees. dead. Otherwise, it will cause me no end of trouble with my Should the defendant’s cross-examination of the man- relatives.’’ Two days later, the landowner asked the friend ager be permitted over the plaintiff’s objection? to return the deed to him because he had decided that he (A) No, because cross-examination should be limited to should devise the land to the friend by will rather than by the subject matter of the direct examination and mat- deed. The friend said that he would destroy the deed and a ters affecting the credibility of the witness. day or so later falsely told the landowner that the deed had (B) No, because the court has not declared the manager been destroyed. Six months ago, the landowner, who had hostile. never executed a will, died intestate, survived by a daugh- (C) Yes, because the cross-examiner is entitled to explore ter as his sole heir at law. The day after the landowner’s matters relevant to any issue in the case, including death, the friend recorded the deed from him. As soon as credibility. the daughter discovered this recording and the friend’s (D) Yes, because the manager is the agent of a party, as to claim to the land, she brought an appropriate action against whom the scope of cross-examination is unlimited. the friend to quiet title to the land. Question 62 For whom should the court hold? In order to combat terrorism, Congress enacted a statute (A) The daughter, because the death of the landowner authorizing the President to construct surveillance facilities deprived the subsequent recordation of any effect. on privately owned property if the President determined that (B) The daughter, because the friend was dishonest in reporting that he had destroyed the deed. the construction of such facilities was ‘‘necessary to safe- guard the security of the United States.’’ The statute pro- (C) The friend, because the deed was delivered to him. vided no compensation for the owner of the land on which (D) The friend, because the deed was recorded by him. such facilities were constructed and provided that the sur- Question 64 veillance facilities were to be owned and operated by the A carpenter contracted with a homeowner to remodel the U.S. government. homeowner’s home for $10,000, to be paid on completion Pursuant to this statute, the President has determined of the work. On May 29, relying on his expectation that that the construction of a surveillance facility on a very he would fi nish the work and have the homeowner’s pay- small, unused portion of an owner’s large tract of land is ment on June 1, the carpenter contracted to buy a car for necessary to safeguard the security of the United States. ‘‘$10,000 in cash, if payment is made on June 1; if payment The construction and operation of the facility will not affect is made thereafter, the price is $12,000.’’ The carpenter any of the uses that the owner is currently making of the completed the work according to specifi cations on June entire tract of land. 1 and demanded payment from the homeowner on that The owner has fi led suit to challenge the constitutional- date. The homeowner, without any excuse, refused to pay. ity of the construction of a surveillance facility on the parcel Thereupon, the carpenter became very excited, suffered of land at issue without compensation. a minor heart attack, and, as a result, incurred medical How should the court rule? expenses of $1,000. The reasonable value of the carpen- (A) It would be a taking of the owner’s property for which ter’s services in remodeling the homeowner’s home was the owner must be compensated. $13,000. (B) It would single out the owner for adverse treatment in In an action by the carpenter against the homeowner, violation of the equal protection component of the Fifth Amendment. which of the following should be the carpenter’s measure (C) It would not interfere with any use the owner is currently of recovery? making of the entire tract of land and, therefore, would (A) $10,000, the contract price. not entitle the owner to any compensation. (B) $11,000, the contract price plus $1,000 for the medical (D) It would be valid without any compensation, because expenses incurred because the homeowner refused to it has been determined to be necessary to protect a pay. compelling government interest in national security. (C) $12,000, the contract price plus $2,000, the bargain that was lost because the carpenter could not pay cash Question 63 for the car on June 1. A landowner executed an instrument in the proper form (D) $13,000, the amount the homeowner was enriched by of a deed, purporting to convey his land to a friend. The the carpenter’s services. Practice MBE—A.M. Questions 419

Question 65 ran to the pedestrian, whom he did not know, and admin- After a liquor store was robbed, the police received an anon- istered fi rst aid, but the pedestrian died in the bystander’s ymous telephone call naming a store employee as the per- arms. The bystander suffered serious emotional distress petrator of the robbery. Honestly believing that their actions as a result of his failure to save the pedestrian’s life, but were permitted by the U.S. Constitution, the police talked he experienced no resulting physical manifestations. The one of the employee’s neighbors into going to the employ- bystander brought a negligence action against the driver. ee’s home with a hidden tape recorder to engage him in a Is the bystander likely to prevail? conversation about the crime. During the conversation, the (A) No, because the bystander assumed the risk. employee admitted committing the robbery. The employee (B) No, because the bystander had no familial or other pre- was charged in state court with the robbery. He moved to existing relationship with the pedestrian. suppress the recording on the grounds that the method of (C) Yes, because danger invites rescue. obtaining it violated his constitutional rights under both the (D) Yes, because the bystander was in the zone of state and federal constitutions. Assume that a clear prece- danger. dent from the state supreme court holds that the conduct of Question 68 the police in making the recording violated the employee’s rights under the state constitution, and that the exclusionary The Federal Automobile Safety Act establishes certain rule is the proper remedy for this violation. safety and performance standards for all automobiles Should the court grant the employee’s motion? manufactured in the United States. The Act creates a fi ve- member ‘‘Automobile Commission’’ to investigate automo- (A) No, because the employee’s federal constitutional rights were not violated, and this circumstance over- bile safety, to make recommendations to Congress for new rides any state constitutional provisions. laws, to make further rules establishing safety and perfor- (B) No, because the police were acting in the good-faith mance standards, and to prosecute violations of the Act. belief that their actions were permitted by the federal The chairman is appointed by the President, two members Constitution. are selected by the president pro tempore of the Senate, (C) Yes, because the making of the recording violated the and two by the speaker of the House of Representatives. state constitution. A minor U.S. car manufacturer seeks to enjoin enforce- (D) Yes, because use of the recording would violate the ment of the Commission’s rules. neighbor’s federal constitutional rights. The best argument that the manufacturer can make is Question 66 that A man who had become very drunk left a bar and started (A) legislative power may not be delegated by Congress to an agency in the absence of clear guidelines to walk home. Another patron of the bar, who had observed (B) the commerce power does not extend to the manufac- the man’s condition, followed him. The patron saw the man ture of automobiles not used in interstate commerce stumble and fall to the ground near an alley. The patron then (C) the manufacturer is denied due process of law because began to pull out a gun but saw that the man had passed it is not represented on the Commission out asleep in the gutter. The patron reached into the man’s (D) the Commission lacks authority to enforce its standards pocket, grabbed his wallet, and started to walk away. When because not all of its members were appointed by the the patron heard police officers approaching, he dropped President of the United States and ran off. The crimes below are listed in descending order of Question 69 seriousness. An uncle was the record title holder of a vacant tract of land. What is the most serious crime for which the patron He often told friends that he would leave the land to his properly could be convicted? nephew in his will. The nephew knew of these conversa- (A) Robbery tions. Prior to the uncle’s death, the nephew conveyed the (B) Larceny land by warranty deed to a woman for $10,000. She did not (C) Attempted Robbery conduct a title search of the land before she accepted the (D) Attempted Larceny deed from the nephew. She promptly and properly recorded her deed. Last month, the uncle died, leaving the land to the Question 67 nephew in his duly probated will. Both the nephew and the A driver negligently ran over a pedestrian. A bystander wit- woman now claim ownership of the land. The nephew has nessed the accident from across the street. The bystander offered to return the $10,000 to the woman. 420 Strategies and Tactics for the Multistate Bar Exam

Who has title to the land? intent to commit a crime therein.’’ Manslaughter is defi ned (A) The nephew, because at the time of the deed to the as the ‘‘killing of a human being in a criminally reckless man- woman, the uncle was the owner of record. ner.’’ Criminal recklessness is ‘‘consciously disregarding a (B) The nephew, because the woman did not conduct a substantial and unjustifi able risk resulting from the actor’s title search. conduct.’’ Murder is defi ned as ‘‘the premeditated and inten- (C) The woman, because of the doctrine of estoppel by tional killing of another or the killing of another in the com- deed. mission of committing rape, robbery, burglary, or arson.’’ (D) The woman, because she recorded her deed prior to Another statute provides that intoxication is not a defense to the uncle’s death. crime unless it negates an element of the offense. The defendant was charged with the murder of the Question 70 watchman and manslaughter in the death of the pedestrian. A mother, whose adult son was a law school graduate, con- Assume that he is tried separately on each charge. tracted with a tutor to give the son a bar exam prepara- At a defendant’s trial for the murder of the watchman, tion course. ‘‘If my son passes the bar exam,’’ the mother the court should in substance charge the jury on the issue explained to the tutor, ‘‘he has been promised a job with a of the defense of intoxication that law fi rm that will pay $55,000 a year.’’ The tutor agreed to (A) intoxication is a defense to the underlying crime of bur- do the work for $5,000, although the going rate is $6,000. glary if the defendant, due to drunkenness, did not form Before the instruction was to begin, the tutor repudiated the an intent to commit a crime within the building, in which contract. Although the mother or the son reasonably could case there can be no conviction for murder unless the have employed, for $6,000, an equally qualifi ed instructor to defendant intentionally and with premeditation killed replace the tutor, neither did so. The son failed the bar exam the watchman. and the law fi rm refused to employ him. It can be shown that (B) voluntary intoxication is not a defense to the crime of had the son received the instruction, he would have passed murder the bar exam. (C) the defendant is guilty of murder despite his intoxica- If the mother and the son join as parties plaintiff and tion only if the state proves beyond a reasonable doubt that the killing of the watchman was premeditated and sue the tutor for breach of contract, how much, if anything, intentional are they entitled to recover? (D) voluntary intoxication is a defense to the crime of mur- (A) $1,000, because all other damages could have der if the defendant would not have killed - been avoided by employing another equally qualifi ed man but for his intoxication instructor. (B) $55,000, because damages of that amount were Question 72 within the contemplation of the parties at the time they A defendant was charged with assault and battery in a contracted. jurisdiction that followed the ‘‘retreat’’ doctrine, and he (C) Nominal damages only, because the mother was not pleaded self-defense. At his trial, the evidence established injured by the breach and the tutor made no promise to the following: a man and his wife were enjoying a drink at the son. a tavern when the defendant entered and stood near the (D) Nothing, because neither the mother nor the son took steps to avoid the consequences of the tutor’s breach. door. The wife whispered to her husband that the defen- dant was the man who had insulted her on the street the Question 71 day before. The husband approached the defendant and A defendant became intoxicated at a bar. He got into his said, ‘‘Get out of here, or I’ll break your nose.’’ The defen- car and drove away. Within a few blocks, craving another dant said, ‘‘Don’t come any closer, or I’ll hurt you.’’ When drink, he stopped his car in the middle of the street, picked the husband raised his fi sts menacingly, the defendant up a brick, and broke the display window of a liquor store. pulled a can of pepper spray from his pocket, aimed it at As he was reaching for a bottle, the night watchman arrived. the husband’s face, and sprayed. The husband fell to the Startled, the defendant turned, and struck the watchman fl oor, writhing in pain. on the head with the bottle, killing him. Only vaguely aware Should the defendant be convicted? of what was happening, the defendant returned to his car, (A) No, because he had no obligation to retreat before consumed more liquor, and then drove off at a high speed. resorting to non-deadly force. Relevant statutes defi ne burglary to include ‘‘break- (B) No, because there is no obligation to retreat when one ing and entering a building not used as a dwelling with the is in an occupied structure. Practice MBE—A.M. Questions 421

(C) Yes, because he failed to retreat even though there was contribute $100,000 to fund an annuity for a clerk who was an opportunity available. a longtime employee of the business. The clerk’s position (D) Yes, because the husband did not threaten to use would be terminated due to the dissolution, and he did not deadly force against him. have a retirement plan. The accountant and the bookkeeper informed the clerk of their plan to fund an annuity for him. Question 73 The clerk, confi dent about his fi nancial future because A defendant was charged with aggravated assault. At trial, of the promised annuity, purchased a retirement home. the victim testifi ed that the defendant beat her savagely, but The accountant later contributed his $100,000 to fund the she was not asked about anything said during the incident. annuity, but the bookkeeper stated that he could afford to The prosecutor then called a witness to testify that when the contribute only $50,000. The accountant agreed that the beating stopped, the victim screamed: ‘‘I’m dying—don’t let bookkeeper should contribute only $50,000. [the defendant] get away with it!’’ Does the clerk have a valid basis for an action against Is the testimony of the witness concerning the victim’s the bookkeeper for the unpaid $50,000? statement admissible? (A) No, because the clerk was bound by the modifi cation (A) No, because it is hearsay not within any exception. of the agreement made by the accountant and the (B) No, because the victim was not asked about the bookkeeper. statement. (B) No, because the clerk was only a donee benefi ciary of (C) Yes, as a statement under belief of impending death, the agreement between the accountant and the book- even though the victim did not die. keeper, and had no vested rights. (D) Yes, as an excited utterance. (C) Yes, because the clerk’s reliance on the promised retirement fund prevented the parties from changing Question 74 the terms. A developer was the owner of a large subdivision. A buyer (D) Yes, because the promises to establish the fund were became interested in purchasing a lot but could not decide made binding by consideration from the clerk’s many between Lot 40 and Lot 41. The price and fair market value years of employment. of each of these two lots was $5,000. The buyer paid the Question 76 developer $5,000, which the developer accepted, and the A report released by a Senate investigating committee developer delivered to the buyer a deed that was properly named three U.S. citizens as helping to organize support executed, complete, and ready for recording in every detail for terrorist activities. All three were employed by the U.S. except that the space in the deed for the lot number was government as park rangers. left blank. The developer told the buyer to fi ll in either Lot Congress enacted a statute naming the three individuals 40 or Lot 41 according to his decision and then record the identifi ed in the report and providing that they could not hold deed. The buyer visited the development the next day and any position of employment with the federal government. completely changed his mind, selecting Lot 25. He fi lled in Which of the following constitutional provisions pro- Lot 25 and duly recorded the deed. The price of Lot 25 and vides the best means for challenging the constitutionality its fair market value was $7,500. of the statute? Immediately upon learning what the buyer had done, the developer brought an appropriate action against the (A) The Bill of Attainder Clause buyer to rescind the transaction. If the developer loses, the (B) The Due Process Clause most likely basis for the judgment is that (C) The Ex Post Facto Clause (A) the developer’s casual business practices created his (D) The Takings Clause loss Question 77 (B) the need for certainty in land title records controls Under the Federal Tort Claims Act, with certain exceptions (C) the agency implied to complete the deed cannot be restricted by the oral understanding not relevant here, the federal government is liable only for (D) the recording of the deed precludes any questioning of negligence. A federally owned and operated nuclear reactor its provisions in its recorded form emitted substantial quantities of radioactive matter that set- tled on a nearby dairy farm, killing the dairy herd and con- Question 75 taminating the soil. At the trial of an action brought against An accountant and a bookkeeper, as part of a contract dis- the federal government by the farm’s owner, the trier of fact solving their accounting business, agreed that each would found that the nuclear plant had a sound design, but that a 422 Strategies and Tactics for the Multistate Bar Exam

valve made by the Acme Engineering Company had mal- (A) No, because improperly failing to stop on the recent functioned and allowed the radioactive matter to escape, occasions does not bear on the plaintiff’s veracity and that Acme Engineering Company is universally regarded as does not contradict his testimony in this case. a quality manufacturer of components for nuclear plants, (B) No, because there is no indication that failing to stop on and that there was no way the federal government could the recent occasions led to convictions. have anticipated or prevented the emission of the radioac- (C) Yes, because improperly failing to stop on the recent occasions bears on the plaintiff’s credibility, since he tive matter. claims to have stopped in this case. If there is no other applicable statute, for whom should (D) Yes, because improperly failing to stop on the recent the trial judge enter judgment? occasions tends to contradict the plaintiff’s claim that (A) The plaintiff, on the ground that the doctrine of res ipsa he was driving carefully at the time he collided with the loquitur applies. defendant. (B) The plaintiff, on the ground that one who allows dan- gerous material to escape to the property of another is Question 80 liable for the damage done. A developer, the owner of a large, undeveloped parcel of (C) The defendant, on the ground that a case under the land, prepared a development plan creating 200 house lots Federal Tort Claims Act has not been proved. in the development with the necessary streets and public (D) The defendant, on the ground that the Acme areas. The plan was fully approved by all necessary govern- Engineering Company is the proximate cause of the mental agencies and duly recorded. However, construction owner’s damage. of the streets, utilities, and other aspects of the development Question 78 of the parcel has not yet begun, and none of the streets can be opened as public ways until they are completed in accor- Driving down a dark road, a defendant accidentally ran over dance with the applicable ordinances in the municipality in a man. The defendant stopped and found that the victim was which the parcel is located. dead. The defendant, fearing that he might be held respon- One of the streets laid out as part of the development sible, took the victim’s wallet, which contained a substantial plan is a border road that abuts an adjacent one-acre parcel amount of money. He removed the identifi cation papers and owned by a widower. The widower’s land has no access to put the wallet and money back into the victim’s pocket. The any public way except an old, poorly developed road, which defendant is not guilty of is inconvenient and cannot be used without great expense. (A) larceny, because he took the papers only to prevent The widower sold his plot to a buyer. The description used identifi cation and not for his own use in the deed from the widower to the buyer was the same (B) larceny, because he did not take anything from a living as that used in prior deeds except that the portion of the victim description that formerly said, ‘‘thence by land of the devel- (C) robbery, because he did not take the papers by means of force or putting in fear oper, north-easterly a distance of 200 feet, more or less,’’ was changed to ‘‘thence by the border road as laid out on (D) robbery, because he did not take anything of monetary value the development plan North 46 degrees East 201.6 feet,’’ with full reference to the plan and its recording data. Question 79 The buyer now seeks a building permit that will show In a civil trial arising from a car accident at an intersection, that he intends to use the border road for access to his land. the plaintiff testifi ed on direct examination that he came to The developer objects to the granting of a building permit on a full stop at the intersection. On cross-examination, the the grounds that he has never granted any right to the wid- defendant’s lawyer asked whether the plaintiff claimed that ower or the buyer to use the border road. There are no gov- he was exercising due care at the time, and the plaintiff erning statutes or ordinances relating to the problem. The replied that he was driving carefully. At a sidebar confer- developer brings an appropriate action in which the right of ence, the defendant’s lawyer sought permission to ask the the buyer to use the border road without an express grant plaintiff about two prior intersection accidents in the last 12 from the developer is at issue. months where he received traffic citations for failing to stop The best argument for the developer in this action is at stop signs. The plaintiff’s lawyer objected. that Should the court allow defense counsel to ask the (A) the buyer’s right must await the action of appropri- plaintiff about the two prior incidents? ate public authorities to open the border road as a Practice MBE—A.M. Questions 423

public street, since no private easements arose by The administrator denied the organization’s request. implication She said that the display of this placard would be offensive (B) the Statute of Frauds prevents the introduction of evi- to the circus, which had paid a substantial sum to place its dence that might prove the necessity for the buyer to placards on the buses, and that she had been told by a cir- use the border road cus employee that none of the photographs on the organiza- (C) the buyer’s right to use the border road is restricted to tion’s placard depicted an animal belonging to this particular the assertion of a way by necessity and the facts pre- circus. Under the relevant city ordinance, the administrator’s clude the success of such a claim decision was fi nal. (D) the buyer would be unjustly enriched if he were permit- The organization sued the administrator in an appropri- ted to use the border road ate court for a declaration that she could not, consistent Question 81 with the First Amendment as made applicable to the states A landowner entered into a single contract with a builder to by the Fourteenth Amendment, refuse to sell the organiza- have three different structures built on separate pieces of tion space for its placard for the reasons she gave. property owned by the landowner. Each structure was dis- Will the organization prevail? tinct from the other two and the parties agreed on a specifi c (A) No, because the administrator’s denial of space to the price for each. After completing the fi rst structure in accor- organization was a reasonable time, manner, and place dance with the terms of the contract, the builder demanded restriction of speech. payment of the specifi ed price for that structure. At the (B) No, because a public official may not allow the use of public facilities for the propagation of a message that same time, the builder told the landowner that the builder he or she believes may create a false or misleading was ‘‘tired of the construction business’’ and would not even impression. begin the other two structures. The landowner refused to (C) Yes, because a public official may not refuse to per- pay anything to the builder. mit the dissemination of a message in a public forum Is the builder likely to prevail in a suit for the agreed wholly on the basis of its content unless that denial is price of the fi rst structure? necessary to serve a compelling government interest. (A) No, because substantial performance is a constructive (D) Yes, because a public official may not refuse to allow condition to the landowner’s duty to pay at the contract the use of any public facility to publish a message deal- rate. ing with an issue of public concern. (B) No, because the builder’s cessation of performance Question 83 without legal excuse is a willful breach of the contract. (C) Yes, because the contract is divisible, and the land- A homeowner and a purchaser entered into a valid, enforce- owner will be required to bring a separate claim for the able, written contract by which the homeowner agreed to builder’s failure to complete the other two structures. sell and the purchaser agreed to purchase the homeowner’s (D) Yes, because the contract is divisible, but the landowner residence. One of the contract provisions was that after clos- will be able to deduct any recoverable damages caused ing, the homeowner had the right to remain in the residence by the builder’s failure to complete the contract. for up to 30 days before delivering possession to the pur- chaser. The closing took place as scheduled. Title passed to Question 82 the purchaser and the homeowner remained in possession. A city owned and operated a municipal bus system. The city Within a few days after the closing, the new house next door, sold space on its buses for the posting of placards. Decisions which was being constructed for the homeowner, burned to on the type of placards that could be posted on the buses the ground, and at the end of the 30-day period after clos- were left wholly to the discretion of the administrator of the ing the homeowner refused to move out of his old house; bus system. Although most of the placards that appeared instead, the homeowner proposed to pay the purchaser a on city buses were commercial advertisements, the admin- monthly rental payment in excess of its fair rental value. istrator had often sold space on the buses for placards pro- The purchaser rejected the proposal and that day brought moting various political, charitable, and religious causes. an appropriate action to gain immediate possession of the A circus bought space on the city buses for placards residence. The contract was silent as to the consequences advertising its forthcoming performances. An animal rights of the homeowner’s failure to give up possession within the organization asked the administrator to sell it space for a 30-day period, and the jurisdiction in which the property is placard with photographs showing the mistreatment of ani- located has no statute dealing directly with this situation, mals in circus shows. although the landlord-tenant law of the jurisdiction requires 424 Strategies and Tactics for the Multistate Bar Exam

a landlord to give a tenant 30 days notice before a tenant (D) improper as a comment on the defendant’s failure to may be evicted. The purchaser did not give the homeowner testify any such 30-day statutory notice. The purchaser’s best legal Question 86 argument in support of his action to gain immediate posses- sion is that the homeowner is a A professor, in a lecture in her psychology course at a pri- vate university, described an experiment in which a group (A) trespasser ab initio of college students in a neighboring city rushed out and (B) licensee washed cars stopped at traffic lights during the rush hour. (C) tenant at sufferance She described how people reacted differently—with shock, (D) tenant from month to month joy, and surprise. At the conclusion of her report, she said, Question 84 ‘‘You understand, of course, that you are not to undertake this or any other experiment unless you fi rst clear it with A producer engaged an inexperienced actress to do a small me.’’ Four of the professor’s students decided to try the role in a new Broadway play for a period of six months at same experiment but did not clear it with the professor. a salary of $200 a week. The actress turned down another One motorist who was a subject of their experiment role in order to accept this engagement. On the third day said, ‘‘I was shocked. There were two people on each side of the run, the actress was hospitalized with infl uenza and of the car. At fi rst I thought negatively. I thought they were a replacement was hired to do the part. A week later, the going to attack me and thought of driving away. Then I qui- actress recovered, but the producer refused to accept eted down and decided there were too many dirty cars in her services for the remainder of the contract period. The the city anyway.’’ actress then brought an action against the producer for Charitable immunity has been abolished in the jurisdic- breach of contract. tion. If the motorist has a valid claim against the students, Which of the following is the actress’s best legal will he also prevail against the university? theory? (A) Yes, if the students would not have performed the (A) Her acting contract with the producer was legally sever- experiment but for the professor’s lecture. able into weekly units. (B) Yes, if the motorist’s claim against the students is based (B) Her performance of the literal terms of the contract was on negligence. physically impossible. (C) No, because the students were not the professor’s (C) Her reliance on the engagement with the producer employees. by declining another acting role created an estoppel against the producer. (D) No, because the professor did not authorize the car washing experiment as a class project. (D) Her failure to perform for one week was not a mate- rial failure so as to discharge the producer’s duty to Question 87 perform. Nine gang members were indicted for the murder of a Question 85 tenth gang member who had become an informant. The A defendant was arrested moments after a forcible rape and gang leader pleaded guilty. At the trial of the other eight, was prosecuted for it. The victim testifi ed she tore the assail- the state’s evidence showed the following: The gang leader ant’s shirt. The defendant did not testify. In jury argument, announced a party to celebrate the recent release of a gang the defendant’s counsel urged that the state’s failure to member from jail. But the party was not what it seemed. The offer in evidence the shirt the defendant was wearing when gang leader had learned that the recently released gang arrested indicated that the evidence would be unfavorable member had earned his freedom by informing the authori- to the state’s case. In his closing argument, the prosecutor ties about the gang’s criminal activities. The gang leader said, ‘‘If the defense had thought the clothing would show decided to use the party to let the other gang members see anything, they could have brought it in as evidence them- what happened to a snitch. He told no one about his plan. selves.’’ The prosecutor’s argument is At the party, after all present had consumed large amounts of liquor, the gang leader announced that the released gang (A) proper as rebuttal to the inference that the evidence would be unfavorable to the prosecution member was an informant and stabbed him with a knife in (B) proper as a comment on the defendant’s failure to front of the others. The eight other gang members watched testify and did nothing while the informant slowly bled to death. (C) improper as an argument going beyond the evidence in The jury found the eight gang members guilty of murder and the case they appealed. Practice MBE—A.M. Questions 425

Should the appellate court uphold the convictions? (C) It is proper to impeach the defendant, but not to prove (A) No, because mere presence at the scene of a crime is that the defendant committed the crime. insufficient to make one an accomplice. (D) It is proper to prove the defendant committed the crime, (B) No, because murder is a specifi c intent crime, and but not to impeach the defendant. there is insufficient evidence to show that they intended Question 90 to kill. (C) Yes, because the gang members made no effort to On March 1, a mechanic contracted to repair a textile manu- save the informant after he had been stabbed. facturer’s knitting machine and to complete the job by March (D) Yes, because voluntary intoxication does not negate 6. On March 2, the manufacturer contracted to produce and criminal responsibility. deliver on March 15 specifi ed cloth to a clothing designer. The manufacturer knew that it would have to use the Question 88 machine then under repair to perform this contract. Because A recently established law school constructed its building the designer’s order was for a rush job, the designer and the in a quiet residential neighborhood. The law school had manufacturer included in their contract a liquidated dam- obtained all of the necessary municipal permits for the ages clause, providing that the manufacturer would pay construction of the building, which included a large clock $5,000 for each day’s delay in delivery after March 15. tower whose clock chimed every hour. The chimes dis- The mechanic was inexcusably fi ve days late in repair- turbed only one homeowner in the neighborhood, who had ing the machine, and, as a result, the manufacturer was purchased her house prior to the construction of the build- fi ve days late in delivering the cloth to the designer. The ing. The homeowner was abnormally sensitive to ringing manufacturer paid $25,000 to the designer as liquidated sounds, such as bells and sirens, and found the chimes to damages and now sues the mechanic for $25,000. Both the be extremely annoying. mechanic and the manufacturer knew when making their In a nuisance action by the homeowner against the law contract on March 1 that under ordinary circumstances the school, will the homeowner prevail? manufacturer would sustain little or no damages of any kind (A) Yes, because the chimes interfere with the homeown- as a result of a fi ve-day delay in the machine repair. er’s use and enjoyment of her property. Assuming that the $5,000 liquidated damages clause (B) Yes, because the homeowner purchased her house in the designer-manufacturer contract is valid, which of prior to the construction of the building. the following arguments will serve as the mechanic’s best (C) No, because the chimes do not disturb the other resi- defense to the manufacturer’s action? dents of the neighborhood. (A) Time was not of the essence in the mechanic-manufac- (D) No, because the law school had the requisite municipal turer contract. permits to erect the clock tower. (B) The mechanic had no reason to foresee on March 1 that the designer would suffer consequential damages Question 89 in the amount of $25,000. A defendant is on trial for attempted fraud. The state charges (C) By entering into the contract with the designer while that the defendant switched a price tag from a cloth coat to knowing that its knitting machine was being repaired, a more expensive fur-trimmed coat and then presented the the manufacturer assumed the risk of any delay loss to latter for purchase at the cash register. The defendant testi- the designer. fi ed in her own behalf that the tag must have been switched (D) In all probability, the liquidated damages paid by the by someone else. On cross-examination, the prosecutor manufacturer to the designer are not the same amount as the actual damages sustained by the designer in asks whether the defendant was convicted on two prior consequence of the manufacturer’s late delivery of the occasions of misdemeanor fraud in the defrauding of a cloth. retailer by the same means of switching the price tag on a fur-trimmed coat. Question 91 Is the question about the convictions for the earlier In response to the need for additional toxic waste landfi lls crimes proper? in a state, the state’s legislature enacted a law authoriz- (A) It is not proper either to impeach the defendant or to ing a state agency to establish fi ve new state-owned and prove that the defendant committed the crime. state-operated toxic waste landfi lls. The law provided that (B) It is proper both to prove that the defendant committed the agency would decide the locations and sizes of the the crime and to impeach the defendant. landfi lls after an investigation of all potential sites and a 426 Strategies and Tactics for the Multistate Bar Exam

determination that the particular sites chosen would not (D) granted, if a person of normal sensitivity would not have endanger public health and would be consistent with the suffered a heart attack under these circumstances. public welfare. Question 93 A community in the state was scheduled for inspec- tion by the agency as a potential toxic waste landfi ll site. Suspecting that students in a dormitory were using nar- Because the community’s residents obtained most of their cotics, the president of a private college arranged for local drinking water from an aquifer that ran under the entire police to place concealed microphones in several suites of community, a citizens’ group, made up of residents of that the dormitory. Using these microphones, the college secu- community, sued the appropriate officials of the agency in rity officers recorded a conversation in which a student federal court. The group sought a declaratory judgment that offered to sell marijuana to a school employee. the selection of the community as the site of a toxic waste was turned over to the local police, who played it for a local landfi ll would be unconstitutional and an injunction prevent- judge. The judge issued a warrant to search the student’s ing the agency from selecting the community as a site for room. The room was searched by police, and marijuana such a landfi ll. The agency officials moved to dismiss. was discovered. Which of the following is the most appropriate basis for The student is charged with unlawful possession of nar- the court to dismiss this suit? cotics. At trial, the student’s motion to prevent the introduc- tion of the marijuana into evidence will most probably be (A) The case presents a non-justiciable political question. (B) The interest of the state in obtaining suitable sites for (A) denied, because the college president, in loco parentis, toxic waste landfi lls is sufficiently compelling to justify had the responsibility of preventing unlawful activity by the selection of the community as a location for such a students under the president’s supervision facility. (B) denied, because there was probable cause to make (C) The Eleventh Amendment bars suits of this kind in the the search and police obtained a warrant before com- federal courts. mencing the search (D) The case is not ripe for a decision on the merits. (C) granted, because the student’s privacy was unreason- ably invaded Question 92 (D) granted, because the electronic surveillance was ‘‘fun- The Rapido is a sports car manufactured by a car com- damentally unfair’’ pany. The Rapido has an excellent reputation for mechani- Question 94 cal reliability with one exception: the motor may stall if the A hospital patient had a heart ailment so serious that his engine has not had an extended warm-up. The plaintiff doctors had concluded that only a heart transplant could had just begun to drive her Rapido in city traffic without a save his life. They therefore arranged to have him fl own to a warm-up when the engine suddenly stalled. A car driven bigger hospital to have the operation performed. by a truck driver rear-ended the plaintiff’s car. The plaintiff The patient’s nephew, who stood to inherit from him, suffered no external physical injuries as a result of the col- poisoned him. The poison produced a reaction that required lision. However, the shock of the crash caused her to suffer postponing the journey. The plane on which the patient was to a severe heart attack. have fl own crashed, and all aboard were killed. By the follow- The plaintiff brought an action against the car company ing day, the patient’s heart was so weakened by the effects of based on strict liability in tort. During the trial, the plaintiff the poison that he suffered a heart attack and died. If charged presented evidence of an alternative engine design of equal with criminal homicide, the nephew should be found cost that would eliminate the stalling problem without impair- ing the functions of the engine in any way. The car company (A) guilty moves for a directed verdict at the close of the evidence. (B) not guilty, because his act did not hasten the deceased’s death, but instead prolonged it by one day This motion should be (C) not guilty, because the deceased was already suffering (A) denied, because the jury could fi nd that an unreason- from a fatal illness ably dangerous defect in the engine was a proximate (D) not guilty, because the poison was not the sole cause cause of the collision. of death (B) denied, if the jury could fi nd that the Rapido was not crashworthy. Question 95 (C) granted, because the truck driver’s failure to stop within an assured clear distance was a superseding cause of A seller, the owner of a tract of land, entered into an enforce- the collision. able written agreement with a buyer providing that the seller Practice MBE—A.M. Questions 427 would sell the tract to the buyer for an agreed price. At the (D) Yes, because the dealer’s statement was intentionally place and time designated for the closing, the seller ten- misleading and the dealer had concealed evidence of dered an appropriate deed, but the buyer responded that he the accident. had discovered a mortgage on the tract and would not com- Question 97 plete the transaction, because the seller’s title was not free of encumbrances, as the contract required. The seller said A manufacturing plant located near a busy highway uses that it was his intent to pay the mortgage from the proceeds and stores highly volatile explosives. The owner of the plant of the sale, and he offered to put the proceeds in escrow has imposed strict safety measures to prevent an explosion for that purpose with any agreeable, responsible escrowee. at the plant. During an unusually heavy windstorm, a large The balance due on the mortgage was substantially less tile was blown off the roof of the plant and crashed into the than the contract purchase price. The buyer refused the windshield of a passing car, damaging it. The driver of the seller’s proposal. The seller began an appropriate legal car brought a strict liability action against the owner of the action against the buyer for specifi c performance. There is plant to recover for the damage to the car’s windshield. no applicable statute in the jurisdiction where the tract is Is the driver likely to prevail? located. The seller’s best legal argument in support of his (A) No, because the damage to the windshield did not claim for relief is that result from the abnormally dangerous aspect of the plant’s activity. (A) as the seller of real estate, he had an implied right (B) No, because the severity of the windstorm was to use the contract proceeds to clear the title being unusual. conveyed (C) Yes, because the plant’s activity was abnormally (B) the lien of the mortgage shifts from the tract to the con- dangerous. tract proceeds (D) Yes, because the plant’s location near a busy highway (C) under the doctrine of equitable conversion, title has was abnormally dangerous. already passed to the buyer and the only issue is how the purchase price is to be allocated Question 98 (D) no provision of the contract has been breached by the seller On January 5, a creditor lent $1,000 to a debtor under a contract calling for the debtor to repay the loan at the rate Question 96 of $100 per month payable on the fi rst day of each month. During negotiations to purchase a used car, a buyer asked On February 1, at the debtor’s request, the creditor agreed a dealer whether the car had ever been in an accident. The to permit payment on February 5. On March 1, the debtor dealer replied: ‘‘It is a fi ne car and has been thoroughly requested a similar time extension and the creditor replied, inspected and comes with a certifi cate of assured quality. ‘‘Don’t bother me each month. Just change the date of pay- Feel free to have the car inspected by your own mechanic.’’ ment to the fi fth of the month. But you must now make the In actuality, the car had been in an accident and the dealer payments by cashier’s check.’’ The debtor said, ‘‘Okay,’’ and had repaired and repainted the car, successfully concealing made payments on March 5 and April 5. On April 6, the evidence of the accident. The buyer declined to have the car creditor sold the loan contract to a bank, but did not tell the inspected by his own mechanic, explaining that he would bank about the agreement permitting payments on the fi fth rely on the dealer’s certifi cate of assured quality. At no time of the month. On April 6, the bank wrote to the debtor: ‘‘Your did the dealer disclose that the car had previously been in debt to [the creditor] has been assigned to us. We hereby an accident. The parties then signed a contract of sale. After inform you that all payments must be made on the fi rst day the car was delivered and paid for, the buyer learned about of the month.’’ the car’s involvement in a major accident. Can the debtor justifi ably insist that the payment date If the buyer sues the dealer to rescind the transaction, for the rest of the installments is the fi fth of each month? is the buyer likely to succeed? (A) No, because a contract modifi cation is not binding on an (A) No, because the buyer had the opportunity to have assignee who had no knowledge of the modifi cation. the car inspected by his own mechanic and declined (B) No, because although the creditor waived the condi- to do so. tion of payment on the fi rst of the month, the bank rein- (B) No, because the dealer did not affirmatively assert that stated it. the car had not been in an accident. (C) Yes, because although the creditor waived the condi- (C) Yes, because the contract was unconscionable. tion of payment on the fi rst of the month, the creditor 428 Strategies and Tactics for the Multistate Bar Exam

could not assign to the bank his right to reinstate that (A) not prevail, because Bill was not an aggressor condition. (B) not prevail unless Bill intended his gesture as a threat (D) Yes, because the creditor could assign to the bank only (C) prevail if the neighbor honestly believed that Bill would those rights the creditor had in the contract at the time attack him of the assignment. (D) prevail only if a reasonable person under the circum- stances would have believed that Bill would attack him Question 99 Al and Bill are identical twins. Al, angry at his neighbor, said, Question 100 ‘‘You’d better stay out of my way. The next time I fi nd you Re-direct examination of a witness must be permitted in around here, I’ll beat you up.’’ Two days later, while in the which of the following circumstances? neighborhood, the neighbor saw Bill coming toward him. As (A) To reply to any matter raised in cross-examination Bill came up to the neighbor, Bill raised his hand. Thinking (B) Only to reply to signifi cant new matter raised in cross- Bill was Al and fearing bodily harm, the neighbor struck examination Bill. (C) Only to reiterate the essential elements of the case If Bill asserts a claim against the neighbor and the (D) Only to supply signifi cant information inadvertently neighbor relies on the privilege of self-defense, the neigh- omitted on direct examination bor will